The laboratory experiment, the most effective method for teaching science, is disappearing from most secondary schoo...

Ceci_Perez on January 14, 2021

Can someone please explain why A is incorrect?

Why is A incorrect?

Reply
Create a free account to read and take part in forum discussions.

Already have an account? log in

shunhe on January 20, 2021

Hi @Ceci_Perez,

Thanks for the question! So this is a weaken question. The conclusion is that “the trend should be stopped.” So we want to weaken that, we want to make it seem less likely that the trend should be stopped, or in other words, show that the trend should continue. ??(A) says that scientific knowledge is changing so rapidly it’s hard for secondary schools to keep up without using computers. Does this weaken? Not really. (A) focuses on computers, but laboratory experiments and computers aren’t shown to be two mutually exclusive things here. Some extra assumptions have to be made to make (A) a good weakener, and generally speaking, we want answer choices that stand well on their own. Remember, we’re looking for the best answer. Comparing (E) to (A), (E) does a better job of weakening.

Hope this helps! Feel free to ask any other questions that you might have.